You are on page 1of 13

PrepTest 73

PrepTest 73
Questions 1–7

Setup:
oo five instrumental pieces: R S T V W
oo the pieces will be sequenced

Conditions:
#1: S — V
#2: R R
T or T
S S

#3: R R
W or W
T T

Overview:
It’s always a good sign when the rules (taken as a whole) include all the active variables. This is a classic
combination. One condition is absolute whereas the other two each have two options. We can explore the
second and third rules further to create a set of four molds. We’ll follow this pattern: first option of rule #2
with the first option of rule #3; first option of rule #2 with the second option of rule #3; second option of
rule #2 with the first option of rule #3; and the second option of rule #2 with the second option of rule #3.

#1 R #2 R W
W T T
S V S V

#3 W R #4 R
T T W
S S
V V
With these molds, we have covered all possibilities, and we can safely move on to the questions.

1. (B) As usual, we’ll just take the individual rules and bounce them off the answer choices to eliminate
the incorrect ones. The first rule allows us to eliminate choice D. The second rule allows us to
eliminate choices A and E. Finally, the third rule eliminates choice C, leaving B as the correct answer.
(A) breaks the second rule
(B) Correct
(C) breaks the third rule
(D) breaks the first rule
(E) breaks the second rule

Logic Games Solutions -1- ©Cambridge LSAT 2014


PrepTest 73 PrepTest 73
2. (C) This could only be true of molds #1 and #2. After placing S fourth, the first rule dictates that we
place V fifth. According to our two molds, there are two possible orderings for the remaining variables:
(1) W, T, and R and (2) T, R, and W.

W T R S V
T R W S V
1 2 3 4 5
(A) As the first solution shows, R need not precede W.
(B) S cannot precede T in this scenario.
(C) Correct. In both solutions, T precedes R.
(D) This cannot be true; V has to be fifth, per the first rule.
(D) As the second solution shows, T can be before W.

3. (B) This could only be true of mold #4. Once we place R first, the only variable that can be second
is S (since it precedes T, W, and V). We’re left with V, T, and W, with the only restriction that T must
precede W (third rule). Now, let’s check the answer choices.

R S
1 2 3 4 5
V, T — W
(A) As diagrammed, S must be second.
(B) Correct. V can be third, which would place T and W fourth and fifth, respectively.

R S V T W
1 2 3 4 5
(C) T must precede W, so W can be fourth at the earliest.
(D) As diagrammed, S must be second.
(E) T can’t be last since it has to come before W.

4. (E) This could only be true of mold #1. The only variable which can T is W, so we’ll place it first.
We’re left with R, S, and V, with the only restriction that S precede V (first rule). Let’s head to the
answer choices.

W T
1 2 3 4 5
R, S — V
(A) As diagrammed, W must be first.
(B) As our diagram shows, W must be first.
(C) V must be preceded by S, so V can’t be third.
(D) As shown, W must be first.
(E) Correct. This would place S and V in the third and fourth slots, respectively.

W T S V R
1 2 3 4 5

Logic Games Solutions -2- ©Cambridge LSAT 2014


PrepTest 73 PrepTest 73
5. (E) Since the question doesn’t provide us with any new information, we’ll have to check the answer
choices against our molds.
(A) Mold #4 is the only one in which R could be first. Since V has to come after S, V couldn’t be
second.
(B) As in choice A, R could only be first in mold #4. However, since W must also follow S and T, W
couldn’t be second.
(C) S could be first in either mold #3 or mold #4. In either case, T is preceded by at least one other
variable, so it couldn’t be second.
(D) T could only be first in mold #2. W couldn’t be second because it must also follow R.
(E) Correct. This could be true of mold #3. We would be left with R, T, and V, with the only
restriction that R precede T (second rule).

W S
1 2 3 4 5
V, R — T

6. (D) This could be true of molds #3 and #4. If we quickly fill in the remaining variables (W, R, and T
for mold #3; R, T, and W for mold #4), we get two viable solutions. Now, on to the choices.

S V W R T
S V R T W
1 2 3 4 5
(A) As shown, S must be first.
(B) As in choice A, we already know that S has to be first.
(C) T must be either fifth or fourth.
(D) Correct. As the first solution shows, R could be fourth.
(E) R must be either third or fourth.

7. (A) This scenario invokes either mold #1 or mold #3. Since we can’t definitively place any of the other
variables, we’ll have to evaluate the answer choices based on what we already know.
(A) Correct. In mold #1, T would have to be second since R, S, and V all follow it. In mold #3, T
has to be fourth or later, since it is preceded by W, R, and S.
(B) This could be true of mold #3. The solution would be as follows.

W S V R T
1 2 3 4 5
(C) Yes, S could be fourth under mold #1 if R preceded it. The solution would be the following.

W T R S V
1 2 3 4 5
(D) This could be true of either mold. Here are the possible solutions.

Logic Games Solutions -3- ©Cambridge LSAT 2014


PrepTest 73 PrepTest 73

W T S V R
W R/S S/R V T
1 2 3 4 5
(E) This could be true of mold #3. We would be left with R, S, and V, with the only restriction that
S precede V (first rule).

W T
1 2 3 4 5
R, S — V

Questions 8–13

Setup:
oo five speakers: L M X Y Z
oo each speaker will give exactly one speech
oo two rooms: G R
oo two speeches at 1 and 2; one speech at 3 (in either G or R)

Conditions:
#1: M — L (same room)
#2: X Z and Y Z

#3: XR XG
LG LR (contrapositive)
ZR ZG

Overview:
This game can be tough to break open at the outset. We only have one definitive ordering rule (the first
one). The second rule tells us orderings that can’t happen; we need to be careful with this, because there
will be “ties,” or two speakers at the same time in this game. At first glance, this rule may not appear to do
very much, but it greatly restricts where we can place Z. Could Z be assigned a 3 P.M. slot? That wouldn’t
work because then it would be preceded by both X and Y. Could Z be assigned a 2 P.M. slot? This would
also be problematic. One of X and Y would have to occupy the remaining 2 slot, and there would be no
way to satisfy the first rule.

X/Y
Z Y/X
1 2 3

Logic Games Solutions -4- ©Cambridge LSAT 2014


PrepTest 73 PrepTest 73
Thus, we know that Z has to be assigned to one of the 1 slots. This inference is huge, and will enable us to
answer the questions efficiently. Here is our final visualization of the game:

G M—L
R Z—X
1 2 3
Z

8. (A) In the usual fashion, we’ll take the rules individually and eliminate incorrect choices. The first rule
eliminates choices D and E. The second rule eliminates choice B, and the third rule eliminates choice
C. This leaves A as the correct answer.
(A) Correct
(B) breaks the second rule
(C) breaks the third rule
(D) breaks the first rule
(E) breaks the first rule

9. (B) This question is designed to be time-consuming, so we’ll have to be smart in moving through
the choices. If you saved this question for last (or next to last), you’d be able to eliminate choice A
(question 10) and choice E (question 12). One thing to keep in mind is that any of the pairs that
don’t include Z would have to be grouped in the 2 slots, since that’s the only place where there will be
sufficient space for them.
(A) There are no obvious violations here. They would have to be paired in the 2 slots. It would force
us to put M and Z into the two 1 slots. X is fairly open, although if M and L are assigned to G,
X would have to be assinged to R (third rule). Notice how we can move past the uncertainty of
the rooms by excluding the letters from this question-specific diagram.

M L
Z Y
1 2 3
(B) Correct. As we deduced, Z has to be assigned to a 1 slot, and L cannot be assigned a 1 slot, due
to the first rule.
(C) This is acceptable. With M in one of the 2 slots, we’d have to place L in the 3 slot, leaving Y and
Z to fill out the two 1 slots.

Y/Z M L
Z/Y X
1 2 3
(D) This doesn’t cause any issues. With X and Y grouped in the 2 slots, M and L must occupy a 1 slot
and a 3 slot, respectively, leaving Z to fill the remaining 1 slot.

Logic Games Solutions -5- ©Cambridge LSAT 2014


PrepTest 73 PrepTest 73

M X L
Z Y
1 2 3
(E) As shown in our diagram for choice C, Y and Z can be at the same time.

10. (A) Since we aren’t given any specific information about the rooms, and the answer choices don’t
address that aspect, we can exclude the room letters for this question. After placing X in the sole 3 slot,
the next question is where we will assign M and L. We can either assign M and L to the same room as
X or the other room. This leaves Z and Y to occupy the remaining slots, in that order. Here’s what we
have so far:

M L X Z Y X
Z Y or M L
1 2 3 1 2 3
(A) Correct. As shown, L and Y have to be in different rooms.
(B) M and X are in the same room in the first scenario.
(C) X and Y are in the same room in the second scenario.
(D) X and Z are in the same room in the second scenario.
(E) This must be true; Y and Z have to be in the same room.

11. (C) Here we have anoher question with no new information. Since this is structured in a similar way
to orientation questions, let’s see if we can use the rules to eliminate incorrect answer choices. The first
rule eliminates choices A and B. In choice A, L precedes M, and in choice B, M and L are not in the
same room. Keep in mind that any variable not listed in the choice must be assigned to R! The second
rule eliminates choice D, since Y precedes Z. Finally, the third rule eliminates choice E since Z is
assigned to G instead of R. This leaves C as the correct answer.
(A) breaks the first rule
(B) breaks the first rule
(C) Correct.
(D) breaks the second rule
(E) breaks the third rule

12. (C) This fills out the two 1 slots. Now, we need to figure out where to place M and L. We can either
put them with Y or with Z, leaving X to occupy the remaining open 2 slot. Here’s what we have so far:

Y M L Y X
Z X or Z M L
1 2 3 1 2 3
(A) As shown, L has to be at 3.
(B) Again, L has to be at 3.
(C) Correct. This could be true of the first scenario.

Logic Games Solutions -6- ©Cambridge LSAT 2014


PrepTest 73 PrepTest 73
(D) As shown, X has to occupy a 2 slot.
(E) Again, X has to occupy a 2 slot.

13. (E) Fortunately, we have a good amount of work upon which we can draw for this question. We must
make sure that the replacement condition limits the game in the same way but doesn’t further limit
things.
(A) Questions 8 and 10 show that L doesn’t have to be at 3.
(B) This choice is tricky. While it’s true that M can’t come before Z, this condition doesn’t ensure
that Y or X can’t come before Z.
(C) This answer choice is tempting. The problem with it is that it doesn’t preclude a situation
in which one of X and Y precedes Z. The original condition precludes both variables from
preceding Z.
(D) As question 10 shows, X can occupy the 3 slot. Also, since X and Y are interchangeable in this
game, it stands to reason that Y could also occupy the 3 slot.
(E) Correct. It’s interesting that the correct answer is a restatement of the game’s major inference.

Questions 14–18

Setup:
oo three families: T W Y
oo five buildings: F G I M S
oo each family owned at least one
oo each building owned by exactly one family

Conditions:
#1: #W > #Y

I M
F F
#2: and

S I
#3: T or Y

Overview:
Since numbers in grouping games are always important, let’s look closely at the first rule. We know that
each family must be assigned at least one of the buildings, so the greatest number of buildings any one
family could be assigned is three. Could Y be assigned two buildings? Per the first rule, W would have to be
assigned three buildings, but that would require six buildings altogether when we only have five. Thus, Y
must be assigned only one building. W must be assigned at least two buildings, again due to the first rule.
This means that T can be assigned two buildings at most. For a basic diagram, this is what we have:

Logic Games Solutions -7- ©Cambridge LSAT 2014


PrepTest 73 PrepTest 73

T W Y
~S I
S ~I

However, we can do more to break this game open. Oftentimes a rule like #3 can be used to split a game
into different scenarios. Since at least one of two things must occur, we’ll develop a scenario for each one
occurring alone and a scenario for them both occurring.

#1 #2 #3
F/M G
S F/M M/F I S M/F I
T W Y T W Y T W Y

Mold #1: When we assign S to T, there’s not anything else to deduce. We can easily satisfy the second rule
by spreading F, I, and M across the three families, so things are wide open. This is the least useful of our
scenarios.
Mold #2: When we assign I to Y, the second rule dictates that we split F and M between T and W. G and
S are pretty wide open in this scenario, as we don’t have any specific restrictions about them.
Mold #3: Finally, a fully fleshed-out mold! When S is assigned to T, and I is assigned to Y, the second rule
dictates that we split F and M between T and W. Since W must have at least two buildings assigned to it,
we can give it G.

14. (A) The game starts off in typical fashion with an orientation question. The first rule eliminates choice
D and choice E. The second rule eliminates choice C. Finally, the third rule eliminates choice B,
leaving A as the correct answer.
(A) Correct
(B) breaks the third rule
(C) breaks the second rule
(D) breaks the first rule
(E) breaks the first rule

15. (D) With no new information, we’ll just check the answer choices against our molds.
(A) This could be true of mold #2 and mold #3.
(B) This could be true of mold #2 and mold #3.
(C) This could be true of mold #1 and mold #2.
(D) Correct. Since T could be assigned a maximum of two buildings, and it’s already assigned one in
each of mold #1 and #2, it couldn’t accommodate both I and M.

Logic Games Solutions -8- ©Cambridge LSAT 2014


PrepTest 73 PrepTest 73
(E) This could be true of mold #1, as shown below. We would have to split F and M between W and
Y (second rule), leaving G to occupy the remaining slot over W.

I G
S F/M M/F
T W Y

16. (D) This could only be true of mold #1. Since we have filled Y’s only slot, we’ll need to split F and I
between T and W (second rule). This leaves G to occupy the remaining W slot. Now, let’s head to the
answer choices.

F/I G
S I/F M
T W Y
(A) This could be true, but T could also be assigned I instead.
(B) Again, T could be assigned either F or I.
(C) W could be assigned I instead of F.
(D) Correct. As our diagram shows, this must be true.
(E) This is the other possibility for W, but it need not be true.

17. (B) The question specifies that G and I will be grouped together, so our natural instinct is to ask which
family could be assigned both of them. This could only happen under mold #1. T can be assigned
a maximum of two buildings, so the GI piece will have to be assigned to W. We don’t know which
building will be assigned to Y, but the only two left are F and M, so it’ll have to be one of them. We
also know that F can’t be assigned to W due to the presence of I (second rule). Let’s check the answers.

G
S I F/M
T W Y
~F
(A) As shown, G is assigned to W.
(B) Correct. F has two options: T or Y.
(C) As we established, this would break the second rule.
(D) S has to be assigned to T in this scenario.
(E) I has to be assigned to W in this scenario.

18. (E) This question addresses a scenario we haven’t yet explored in much detail: one in which W is
assigned three buildings. We do have a previous diagram that we can use here. As question 17 proves,
T can be assigned S alone without violating any of the conditions. In that case, W would get M, and
Y would get F. Thus, choices A, B, and C can all be eliminated, since none of them include S. Let’s
test answer choice C, and see if I can be the only building assigned to T. A quick glance at our molds

Logic Games Solutions -9- ©Cambridge LSAT 2014


PrepTest 73 PrepTest 73
indicates that T must be assigned either S or one of F and M, so it isn’t possible for T to be assigned
just I. By process of elimination, answer choice E is correct.

Questions 19–23

Setup:
oo three bouquets: 1 2 3
oo each bouquet will have one or more kinds of flower
oo flower kinds: L P R S T

Conditions:
#1: 1 and 3 have no kinds in common
#2: 2 and 3 have exactly two kinds in common

S
#3: 3
#4: R ~R
L ~L (contrapositive)
~S S
#5: T P; ~P ~T (contrapositive)

Overview:
This game has a couple of rules (1 and 2) which are fairly tricky to diagram. They point to the importance
of establishing some numerical limitations at the outset. The second rule dictates that bouquets 2 and 3
must each have at least two kinds of flowers. Because we know L and S can’t be in the same bouquet, the
maximum number of flower kinds per bouquet across the board is four. Now let’s examine the numbers
for bouquet 1. It can’t contain S due to rules 1 and 3. Since we know that bouquet 3 contains at least two
kinds of flowers (second rule), there must be another kind that bouquet 1 can’t have, so at maximum,
it can contain three kinds of flowers. An important thing to note with this game is that an acceptable
solution does not have to include all five kinds of flowers. If that was a requirement, it would most likely
have been mentioned in the setup. For our diagram, we’ll put 1 and 3 next to each other since there’s no
ordering component to the game and it makes it easier to represent the first rule.

= L T
= R ~S P
1 ≠ 3 2
~S S

Logic Games Solutions -10- ©Cambridge LSAT 2014


PrepTest 73 PrepTest 73
19. (A) As usual, we’ll use the rules to eliminate incorrect choices. The first rule eliminates choice B. All
the choices satisfy the second rule, so it’s not very helpful here. The third rule eliminates choice E. The
fourth rule eliminates choice D and the fifth rule eliminates choice C, leaving A as the correct answer.
(A) Correct
(B) breaks the first rule
(C) breaks the fifth rule
(D) breaks the fourth rule
(E) breaks the third rule

20. (B) This scenario triggers the fourth rule, so we know that bouquet 1 is also assigned R. Next, the first
rule tells us that bouquet 3 will not have L or R. Bouquet 3 must have at least two kinds of flowers, so
the natural question is which ones. The options are P, S, and T. This can be a tough inference to see,
but we have a conditional about T and P (the fifth rule). The contrapositive tells us that if a bouquet
doesn’t have P, then it doesn’t have T. To avoid triggering this contrapositive, which would leave it with
only one kind of flower, bouquet 3 must have P. Note that 1 can only have two kinds in this case; it
can’t have T since the fifth rule would force it to also have P, it can’t have P since Bouquet 3 does, and
it can’t have S due to the first and third rules. Bouquet 3 can have a maximum of three kinds since 1
has at two kinds (L and R), and the first rule precludes them from sharing any kinds. Bouquet 2 must
get at least oen of P and S to satisfy the second rule. Here’s where our diagram stands:

R P
L S P/S
1 ≠ 3 2
~S ~L
~R
(A) We don’t know very much about bouquet 2, and it certainly need not be assigned L.
(B) Correct. As we deduced 3 must be assigned a P.
(C) As in choice A, we have very little information about bouquet 2.
(D) Again, we can’t know this for sure, but it is possible.
(E) This doesn’t have to be true, although it could be.

21. (E) This triggers the fifth rule, and bouquet 1 must also have P. We’ll next turn our attention to
bouquet 3, since the two are closely related. T and P are both out for bouquet 3 due to the first rule.
We already know that bouquet 3 gets an S (third rule). The only other options for bouquet 3 are L and
R. Since triggering the contrapositive of the fourth rule would leave bouquet 3 with only one kind of
flower, we have to avoid that, and give 3 an R. To satisfy the second rule, bouquet 2 must have at least
one of R and S. Further, if bouquet 2 doesn’t have both R and S, the second rule would force us to
assign it an L. Bouquet 1 is limited to just T and P, since assigning it an L would also force it to have
an R (fourth rule). Now, on to the choices!

Logic Games Solutions -11- ©Cambridge LSAT 2014


PrepTest 73 PrepTest 73

T R
P S R/S
1 ≠ 3 2
~S ~P
~T
(A) This choice doesn’t include any of L, R, and S, so it’s out!
(B) This only includes S, so there’d be no way to satisfy the second rule.
(C) As in choice B, we only have S, so the second rule would be broken.
(D) This choice only has an R; without one of L and S, we couldn’t satisfy the second rule.
(E) Correct. This choice has both R and S, so the second rule can be satisfied. Since 2 and 3 share
exactly two kinds, 3 would be limited to just R and S.

P
T
T R = R
P S = S
1 3 2

22. (A) The second rule is the only one that specifically addresses bouquet 3, so we need to be on high
alert for a choice which violates it. With no additional information, we’ll go straight to the choices.
(A) Correct. What do we know about bouquets that include L? They don’t have S (fourth rule). The
problem with this is that bouquet 3 would be limited to L and R (second rule), which would in
turn break the third rule.

L = L
R = R
1 ≠ 3 2
~S S
(B) This is acceptable. Bouquet 3 could have S, P, and T, satisfying the second rule.
(C) This works too. In addition to S, bouquet 3 would have one of P and R.
(D) This is acceptable. On top of S, bouquet 3 would have two of P, R, and T. Since leaving P out
would break the second rule (due to the fifth), at least one of bouquet 3’s flower kinds would be
P.
(E) This scenario is fine. Bouquet 3 would have one of P and R in addition to S. Bouquet 3 couldn’t
be assigned a T because it would trigger the fifth condition and bring P along, thus giving 2 and

Logic Games Solutions -12- ©Cambridge LSAT 2014


PrepTest 73 PrepTest 73
3 three flower kinds in common (P, S, and T).

23. (C) Questions like this can eat up a lot of time, so fortunately, it’s the last in the set. With no new
information, we’ll head straight to the choices.
(A) This was true of our diagram for question 20, so we’ll eliminate it.
(B) Our diagram for question 21 disproves this choice, so let’s keep going.
(C) Correct. Bouquet 3 must include S (third rule), and it would also have to include P and R due
to the second rule. Then, we’d have to consider what could be assigned to 1. S, P, and R are all
out because they’re assigned to 3. The only other possibilities are L and T. If L were assigned to
1, it would bring along R (fourth rule) and break the first rule. If T were assigned to 1, it would
bring along P (fifth rule) and break the first rule.

S L
P P
R R
1 ≠ 3 2
(D) Our diagram for question 21 choice E shows that this is acceptable. Taking away the T assigned
to 2 wouldn’t affect the validity of the solution.
(E) We don’t have a scenario that matches this one, but it isn’t hard to make one. Let’s assign T and P
to bouquet 3. We can then assign R to bouquet 1 without breaking any rules.

P
S = S
R T = T
1 3 2

Logic Games Solutions -13- ©Cambridge LSAT 2014

You might also like